Does f(x) have to be in the ternary Cantor set?

  • Thread starter Thread starter mkkrnfoo85
  • Start date Start date
  • Tags Tags
    Cantor Proof Set
Click For Summary
The discussion revolves around proving that the function f(x) = (x+2)/3 is a contraction mapping that maps the ternary Cantor set into itself. The fixed point of the function is identified as 1, and the contraction property is established with a constant q < 1. A key point raised is the interpretation of mapping the Cantor set, which means that if x is in the Cantor set, then f(x) must also be in the Cantor set. The clarification provided highlights that a number is in the Cantor set if its base 3 expansion contains no 1s, and the function's behavior with respect to intervals confirms that it indeed maps the Cantor set into itself. Overall, the function's properties align with the characteristics of the ternary Cantor set.
mkkrnfoo85
Messages
50
Reaction score
0
Hey all,

I would really like help on this probably simple proof:

That the map x |--> f(x) = (x+2)/3 on [0,1] is a contraction,
and maps the ternary Cantor set into itself. Also, find it's fixed point.


(1) I can easily show the fixed point (where f(x) = x) is 1.
(2) I can also pretty easily show it is a contraction:
where |f(x) - xo| <= q*|x - xo|, where q < 1, and xo is the fixed point.

(3) However, I can't seem to find a way to tell whether it maps the ternary Cantor set into itself. I kno the definition of the ternary Cantor set is taking the interval [0,1] and deleting the middle-third of the interval, and then repeating the process on each remaining interval, infinitely.

What does it mean by mapping the ternary Cantor set into itself? Does x have to start out being in the ternary Cantor set? If so, how is it possible if x can vary from [0,1]? What am I interpreting wrong?

Thanks,

Mark
 
Physics news on Phys.org
Look at how the function maps certain intervals. What sort of intervals must x be contained in for it to be in the Cantor set? What then happens to the intervals under function f(x)?
 
ah thx. just the boost i needed =)
 
What does it mean by mapping the ternary Cantor set into itself?

It means that if x is in the subset of [0,1] called the Cantor set, then f(x) is also in the Cantor set. You might use the fact that x is in the Cantor set if and only if its base 3 expansion contains no 1s.

(I've edited to change "contains no 0s" to "contains no 1s"!)

I've also given a little more thought to this. Since dividing by 3 base 3 just shifts the "decimal" point, this is trivial!
 
Last edited by a moderator:
Question: A clock's minute hand has length 4 and its hour hand has length 3. What is the distance between the tips at the moment when it is increasing most rapidly?(Putnam Exam Question) Answer: Making assumption that both the hands moves at constant angular velocities, the answer is ## \sqrt{7} .## But don't you think this assumption is somewhat doubtful and wrong?

Similar threads

  • · Replies 1 ·
Replies
1
Views
3K
  • · Replies 4 ·
Replies
4
Views
2K
  • · Replies 12 ·
Replies
12
Views
2K
  • · Replies 2 ·
Replies
2
Views
2K
  • · Replies 2 ·
Replies
2
Views
8K
  • · Replies 2 ·
Replies
2
Views
1K
  • · Replies 24 ·
Replies
24
Views
4K
  • · Replies 23 ·
Replies
23
Views
7K
  • · Replies 7 ·
Replies
7
Views
7K
  • · Replies 2 ·
Replies
2
Views
3K